Finding the Critical points of a system












0












$begingroup$


Find all critical points of the system



$y_1'= y_1(10-y_1-y_2)$



$y_2'= y_2(30-2y_1-y_2)$



then classify them as stable, asymptotically stable, or unstable.



I need help with this particular question, as you may see, the only problem is that I have NO idea how to determine the critical points of the system , nonetheless, I have a thought, if the critical points are the eigenvalues of the system, then can't I turn this into a matrix and determine the eigenvalues from the matrix, or does this not work? How would I go about determining these critical points as eigenvalues. Any suggestion of literature that may help or explanations would be immensely appreciated.










share|cite|improve this question









$endgroup$

















    0












    $begingroup$


    Find all critical points of the system



    $y_1'= y_1(10-y_1-y_2)$



    $y_2'= y_2(30-2y_1-y_2)$



    then classify them as stable, asymptotically stable, or unstable.



    I need help with this particular question, as you may see, the only problem is that I have NO idea how to determine the critical points of the system , nonetheless, I have a thought, if the critical points are the eigenvalues of the system, then can't I turn this into a matrix and determine the eigenvalues from the matrix, or does this not work? How would I go about determining these critical points as eigenvalues. Any suggestion of literature that may help or explanations would be immensely appreciated.










    share|cite|improve this question









    $endgroup$















      0












      0








      0





      $begingroup$


      Find all critical points of the system



      $y_1'= y_1(10-y_1-y_2)$



      $y_2'= y_2(30-2y_1-y_2)$



      then classify them as stable, asymptotically stable, or unstable.



      I need help with this particular question, as you may see, the only problem is that I have NO idea how to determine the critical points of the system , nonetheless, I have a thought, if the critical points are the eigenvalues of the system, then can't I turn this into a matrix and determine the eigenvalues from the matrix, or does this not work? How would I go about determining these critical points as eigenvalues. Any suggestion of literature that may help or explanations would be immensely appreciated.










      share|cite|improve this question









      $endgroup$




      Find all critical points of the system



      $y_1'= y_1(10-y_1-y_2)$



      $y_2'= y_2(30-2y_1-y_2)$



      then classify them as stable, asymptotically stable, or unstable.



      I need help with this particular question, as you may see, the only problem is that I have NO idea how to determine the critical points of the system , nonetheless, I have a thought, if the critical points are the eigenvalues of the system, then can't I turn this into a matrix and determine the eigenvalues from the matrix, or does this not work? How would I go about determining these critical points as eigenvalues. Any suggestion of literature that may help or explanations would be immensely appreciated.







      ordinary-differential-equations proof-writing






      share|cite|improve this question













      share|cite|improve this question











      share|cite|improve this question




      share|cite|improve this question










      asked Dec 13 '18 at 12:10









      lastgunslingerlastgunslinger

      728




      728






















          2 Answers
          2






          active

          oldest

          votes


















          0












          $begingroup$

          The stationary or critical points are the points where the solutions through them are constant. Which means that the derivatives are zero.



          In $y_1'=0=y_2'$ you get trivially $(y_1,y_2)=(0,0)$ as solution, then setting only one component to zero $(y_1,y_2)=(0,30)$ and $(y_1,y_2)=(10,0)$ and then as fourth point the solution of the linear system of the second factors.






          share|cite|improve this answer









          $endgroup$





















            0












            $begingroup$

            Critical points are points where $(y'_1, y_2') = (0, 0)$, so in your case these are the possible cases




            $y_1 = 0$ and $y_2 = 0$




            Let's call that point ${bf x}_1 = (0, 0)$




            $y_1 = 0$ and $30−2y_1−y_2 = 0$




            This leads to $y_2 = 30$, let's call that point ${bf x}_2 = (0, 30)$




            $10−y_1−y_2 = 0$ and $y_2 = 0$




            This leads to $y_1 = 10$, let's call that point ${bf x}_3 = (10, 0)$




            $10−y_1−y_2 = 0$ and $30−2y_1−y_2 = 0$




            In this case the solution is ${bf x}_4 = (20, -10)$



            To test the stability, you just need to calculate the Jacobian matrix and evaluate its eigenvalues at each of the four locations above.



            $$
            J = pmatrix{partial y_1'/partial y_1 & partial y_1'/partial y_2 \
            partial y_2'/partial y_1 & partial y_2'/partial y_2}
            $$



            The plot below will help you confirm that the numbers you calculate for the eigenvalues make sense



            enter image description here






            share|cite|improve this answer











            $endgroup$













              Your Answer





              StackExchange.ifUsing("editor", function () {
              return StackExchange.using("mathjaxEditing", function () {
              StackExchange.MarkdownEditor.creationCallbacks.add(function (editor, postfix) {
              StackExchange.mathjaxEditing.prepareWmdForMathJax(editor, postfix, [["$", "$"], ["\\(","\\)"]]);
              });
              });
              }, "mathjax-editing");

              StackExchange.ready(function() {
              var channelOptions = {
              tags: "".split(" "),
              id: "69"
              };
              initTagRenderer("".split(" "), "".split(" "), channelOptions);

              StackExchange.using("externalEditor", function() {
              // Have to fire editor after snippets, if snippets enabled
              if (StackExchange.settings.snippets.snippetsEnabled) {
              StackExchange.using("snippets", function() {
              createEditor();
              });
              }
              else {
              createEditor();
              }
              });

              function createEditor() {
              StackExchange.prepareEditor({
              heartbeatType: 'answer',
              autoActivateHeartbeat: false,
              convertImagesToLinks: true,
              noModals: true,
              showLowRepImageUploadWarning: true,
              reputationToPostImages: 10,
              bindNavPrevention: true,
              postfix: "",
              imageUploader: {
              brandingHtml: "Powered by u003ca class="icon-imgur-white" href="https://imgur.com/"u003eu003c/au003e",
              contentPolicyHtml: "User contributions licensed under u003ca href="https://creativecommons.org/licenses/by-sa/3.0/"u003ecc by-sa 3.0 with attribution requiredu003c/au003e u003ca href="https://stackoverflow.com/legal/content-policy"u003e(content policy)u003c/au003e",
              allowUrls: true
              },
              noCode: true, onDemand: true,
              discardSelector: ".discard-answer"
              ,immediatelyShowMarkdownHelp:true
              });


              }
              });














              draft saved

              draft discarded


















              StackExchange.ready(
              function () {
              StackExchange.openid.initPostLogin('.new-post-login', 'https%3a%2f%2fmath.stackexchange.com%2fquestions%2f3037921%2ffinding-the-critical-points-of-a-system%23new-answer', 'question_page');
              }
              );

              Post as a guest















              Required, but never shown

























              2 Answers
              2






              active

              oldest

              votes








              2 Answers
              2






              active

              oldest

              votes









              active

              oldest

              votes






              active

              oldest

              votes









              0












              $begingroup$

              The stationary or critical points are the points where the solutions through them are constant. Which means that the derivatives are zero.



              In $y_1'=0=y_2'$ you get trivially $(y_1,y_2)=(0,0)$ as solution, then setting only one component to zero $(y_1,y_2)=(0,30)$ and $(y_1,y_2)=(10,0)$ and then as fourth point the solution of the linear system of the second factors.






              share|cite|improve this answer









              $endgroup$


















                0












                $begingroup$

                The stationary or critical points are the points where the solutions through them are constant. Which means that the derivatives are zero.



                In $y_1'=0=y_2'$ you get trivially $(y_1,y_2)=(0,0)$ as solution, then setting only one component to zero $(y_1,y_2)=(0,30)$ and $(y_1,y_2)=(10,0)$ and then as fourth point the solution of the linear system of the second factors.






                share|cite|improve this answer









                $endgroup$
















                  0












                  0








                  0





                  $begingroup$

                  The stationary or critical points are the points where the solutions through them are constant. Which means that the derivatives are zero.



                  In $y_1'=0=y_2'$ you get trivially $(y_1,y_2)=(0,0)$ as solution, then setting only one component to zero $(y_1,y_2)=(0,30)$ and $(y_1,y_2)=(10,0)$ and then as fourth point the solution of the linear system of the second factors.






                  share|cite|improve this answer









                  $endgroup$



                  The stationary or critical points are the points where the solutions through them are constant. Which means that the derivatives are zero.



                  In $y_1'=0=y_2'$ you get trivially $(y_1,y_2)=(0,0)$ as solution, then setting only one component to zero $(y_1,y_2)=(0,30)$ and $(y_1,y_2)=(10,0)$ and then as fourth point the solution of the linear system of the second factors.







                  share|cite|improve this answer












                  share|cite|improve this answer



                  share|cite|improve this answer










                  answered Dec 13 '18 at 12:21









                  LutzLLutzL

                  58k42054




                  58k42054























                      0












                      $begingroup$

                      Critical points are points where $(y'_1, y_2') = (0, 0)$, so in your case these are the possible cases




                      $y_1 = 0$ and $y_2 = 0$




                      Let's call that point ${bf x}_1 = (0, 0)$




                      $y_1 = 0$ and $30−2y_1−y_2 = 0$




                      This leads to $y_2 = 30$, let's call that point ${bf x}_2 = (0, 30)$




                      $10−y_1−y_2 = 0$ and $y_2 = 0$




                      This leads to $y_1 = 10$, let's call that point ${bf x}_3 = (10, 0)$




                      $10−y_1−y_2 = 0$ and $30−2y_1−y_2 = 0$




                      In this case the solution is ${bf x}_4 = (20, -10)$



                      To test the stability, you just need to calculate the Jacobian matrix and evaluate its eigenvalues at each of the four locations above.



                      $$
                      J = pmatrix{partial y_1'/partial y_1 & partial y_1'/partial y_2 \
                      partial y_2'/partial y_1 & partial y_2'/partial y_2}
                      $$



                      The plot below will help you confirm that the numbers you calculate for the eigenvalues make sense



                      enter image description here






                      share|cite|improve this answer











                      $endgroup$


















                        0












                        $begingroup$

                        Critical points are points where $(y'_1, y_2') = (0, 0)$, so in your case these are the possible cases




                        $y_1 = 0$ and $y_2 = 0$




                        Let's call that point ${bf x}_1 = (0, 0)$




                        $y_1 = 0$ and $30−2y_1−y_2 = 0$




                        This leads to $y_2 = 30$, let's call that point ${bf x}_2 = (0, 30)$




                        $10−y_1−y_2 = 0$ and $y_2 = 0$




                        This leads to $y_1 = 10$, let's call that point ${bf x}_3 = (10, 0)$




                        $10−y_1−y_2 = 0$ and $30−2y_1−y_2 = 0$




                        In this case the solution is ${bf x}_4 = (20, -10)$



                        To test the stability, you just need to calculate the Jacobian matrix and evaluate its eigenvalues at each of the four locations above.



                        $$
                        J = pmatrix{partial y_1'/partial y_1 & partial y_1'/partial y_2 \
                        partial y_2'/partial y_1 & partial y_2'/partial y_2}
                        $$



                        The plot below will help you confirm that the numbers you calculate for the eigenvalues make sense



                        enter image description here






                        share|cite|improve this answer











                        $endgroup$
















                          0












                          0








                          0





                          $begingroup$

                          Critical points are points where $(y'_1, y_2') = (0, 0)$, so in your case these are the possible cases




                          $y_1 = 0$ and $y_2 = 0$




                          Let's call that point ${bf x}_1 = (0, 0)$




                          $y_1 = 0$ and $30−2y_1−y_2 = 0$




                          This leads to $y_2 = 30$, let's call that point ${bf x}_2 = (0, 30)$




                          $10−y_1−y_2 = 0$ and $y_2 = 0$




                          This leads to $y_1 = 10$, let's call that point ${bf x}_3 = (10, 0)$




                          $10−y_1−y_2 = 0$ and $30−2y_1−y_2 = 0$




                          In this case the solution is ${bf x}_4 = (20, -10)$



                          To test the stability, you just need to calculate the Jacobian matrix and evaluate its eigenvalues at each of the four locations above.



                          $$
                          J = pmatrix{partial y_1'/partial y_1 & partial y_1'/partial y_2 \
                          partial y_2'/partial y_1 & partial y_2'/partial y_2}
                          $$



                          The plot below will help you confirm that the numbers you calculate for the eigenvalues make sense



                          enter image description here






                          share|cite|improve this answer











                          $endgroup$



                          Critical points are points where $(y'_1, y_2') = (0, 0)$, so in your case these are the possible cases




                          $y_1 = 0$ and $y_2 = 0$




                          Let's call that point ${bf x}_1 = (0, 0)$




                          $y_1 = 0$ and $30−2y_1−y_2 = 0$




                          This leads to $y_2 = 30$, let's call that point ${bf x}_2 = (0, 30)$




                          $10−y_1−y_2 = 0$ and $y_2 = 0$




                          This leads to $y_1 = 10$, let's call that point ${bf x}_3 = (10, 0)$




                          $10−y_1−y_2 = 0$ and $30−2y_1−y_2 = 0$




                          In this case the solution is ${bf x}_4 = (20, -10)$



                          To test the stability, you just need to calculate the Jacobian matrix and evaluate its eigenvalues at each of the four locations above.



                          $$
                          J = pmatrix{partial y_1'/partial y_1 & partial y_1'/partial y_2 \
                          partial y_2'/partial y_1 & partial y_2'/partial y_2}
                          $$



                          The plot below will help you confirm that the numbers you calculate for the eigenvalues make sense



                          enter image description here







                          share|cite|improve this answer














                          share|cite|improve this answer



                          share|cite|improve this answer








                          edited Dec 13 '18 at 12:29

























                          answered Dec 13 '18 at 12:24









                          caveraccaverac

                          14.6k31130




                          14.6k31130






























                              draft saved

                              draft discarded




















































                              Thanks for contributing an answer to Mathematics Stack Exchange!


                              • Please be sure to answer the question. Provide details and share your research!

                              But avoid



                              • Asking for help, clarification, or responding to other answers.

                              • Making statements based on opinion; back them up with references or personal experience.


                              Use MathJax to format equations. MathJax reference.


                              To learn more, see our tips on writing great answers.




                              draft saved


                              draft discarded














                              StackExchange.ready(
                              function () {
                              StackExchange.openid.initPostLogin('.new-post-login', 'https%3a%2f%2fmath.stackexchange.com%2fquestions%2f3037921%2ffinding-the-critical-points-of-a-system%23new-answer', 'question_page');
                              }
                              );

                              Post as a guest















                              Required, but never shown





















































                              Required, but never shown














                              Required, but never shown












                              Required, but never shown







                              Required, but never shown

































                              Required, but never shown














                              Required, but never shown












                              Required, but never shown







                              Required, but never shown







                              Popular posts from this blog

                              Quarter-circle Tiles

                              build a pushdown automaton that recognizes the reverse language of a given pushdown automaton?

                              Mont Emei